Maximizing the volume of a cone formed by revolving a right triangle

In summary, the triangle hypotenuse given rectangle is rotated around one of their legs to generate a right circular cone. The objective function is to maximize the volume of the cone, and the constraint is that the radius and height must be related by the Pythagorean theorem. The maximum volume is found when the height is equal to the radius divided by 3, and the cone has a slant height of $L$.
  • #1
leprofece
241
0
A triangle hypotenuse given rectangle is rotated around one of their legs to generate a right circular cone?
find the cone of greater volume.
resp V= (2Sqrt(3)pi L^3)/27

It says hypotenuse given but it has no value According to the answer you can name it L
 
Physics news on Phys.org
  • #2
Re: max and min 3

Hello leprofece,

We require that you show your work, as I have stated once before. Our desire is to help you solve the problem, rather than work it for you. This way you learn more, by being an active participant in the process of solving the problem.

To being the problem, can you state the objective function, that is, the function you wish to optimize, along with the constraint on the variables?

The objective function will be the volume of a right circular cone, and the constraint will be the relationship between the radius and height of the cone with its slant height, which you have labeled as $L$.
 
  • #3
Re: max and min 3

MarkFL said:
Hello leprofece,

We require that you show your work, as I have stated once before. Our desire is to help you solve the problem, rather than work it for you. This way you learn more, by being an active participant in the process of solving the problem.

To being the problem, can you state the objective function, that is, the function you wish to optimize, along with the constraint on the variables?

The objective function will be the volume of a right circular cone, and the constraint will be the relationship between the radius and height of the cone with its slant height, which you have labeled as $L$.

R+ H =L

is that right??'

- - - Updated - - -

leprofece said:
R2+ H2 =L2

is that right??'
R+ H =L
 
  • #4
Re: max and min 3

Yes, the two legs of the right triangle are the radius and height of the resulting cone, and they are indeed related to the slant height (the hypotenuse of the right triangle) by the Pythagorean theorem:

\(\displaystyle R^2+H^2=L^2\)

So this is the constraint...what is the objective function, i.e., the volume of the cone?
 
  • #5
Re: max and min 3

markfl said:
yes, the two legs of the right triangle are the radius and height of the resulting cone, and they are indeed related to the slant height (the hypotenuse of the right triangle) by the pythagorean theorem:

\(\displaystyle r^2+h^2=l^2\)

so this is the constraint...what is the objective function, i.e., the volume of the cone?

r2= l2-h2
 
  • #6
Re: max and min 3

leprofece said:
r2= l2-h2

the V pi(L2-h2[/QUOTE]*H/3and derive with respect to L
L2h-h2[/QUOTE]
hL2-h^3/3
derivating I got 2hL-3h^2/3
and This is equal to L =3h2/(2h)
That is right isnot it?
 
  • #7
Re: max and min 3

Your objective function is the volume of the cone:

\(\displaystyle V=\frac{1}{3}\pi R^2H\)

Using:

\(\displaystyle R^2=L^2-H^2\)

This becomes:

\(\displaystyle V(H)=\frac{1}{3}\pi \left(L^2-H^2 \right)H=\frac{1}{3}\pi\left(HL^2-H^3 \right)\)

Differentiating with respect to $H$ and equating the result to zero, we obtain:

\(\displaystyle V'(H)=\frac{1}{3}\pi\left(L^2-3H^2 \right)=0\)

Recall $L$ is a constant.

This implies:

\(\displaystyle L^2-3H^2=0\implies H=\frac{L}{\sqrt{3}}\)

We observe that the second derivative of the volume function is negative at this critical value which means we have a maximum. And so we may conclude:

\(\displaystyle V_{\max}=V\left(\frac{L}{\sqrt{3}} \right)=\frac{1}{3}\pi\left(\left(\frac{L}{\sqrt{3}} \right)L^2-\left(\frac{L}{\sqrt{3}} \right)^3 \right)=\frac{1}{3}\pi \frac{L^3}{\sqrt{3}}\left(1-\frac{1}{3} \right)=\frac{2\sqrt{3}}{27}\pi L^3\)
 

Related to Maximizing the volume of a cone formed by revolving a right triangle

What is the formula for calculating the volume of a cone formed by revolving a right triangle?

The formula for calculating the volume of a cone formed by revolving a right triangle is V = (1/3)πr²h, where r is the radius of the base and h is the height of the cone.

How do you find the radius and height of the cone from a right triangle?

The radius of the cone is equal to the length of the hypotenuse of the right triangle. The height of the cone can be found using the Pythagorean theorem, where h = √(l² - r²), where l is the length of one of the legs of the right triangle.

What is the relationship between the volume of the cone and the dimensions of the right triangle?

The volume of the cone formed by revolving a right triangle is directly proportional to the length of the hypotenuse and the height of the right triangle. This means that if the length or height of the triangle is increased, the volume of the cone will also increase.

Can the volume of the cone be maximized?

Yes, the volume of the cone can be maximized by finding the dimensions of the right triangle that will result in the largest possible volume. This can be achieved by using calculus to find the maximum value of the volume function.

What are some real-life applications of maximizing the volume of a cone formed by revolving a right triangle?

This concept is commonly used in architecture and engineering to determine the optimal dimensions for structures such as chimneys, silos, and cooling towers. It is also used in manufacturing to create efficient and space-saving designs for containers and packaging. Additionally, understanding this concept is important in fields like physics and astronomy, where the volume of cones is used to calculate the volume of objects such as planets and stars.

Similar threads

  • Calculus and Beyond Homework Help
Replies
8
Views
1K
Replies
2
Views
2K
  • Calculus and Beyond Homework Help
Replies
4
Views
1K
  • Calculus and Beyond Homework Help
Replies
3
Views
959
Replies
6
Views
2K
Replies
6
Views
2K
  • General Math
Replies
5
Views
2K
  • Calculus and Beyond Homework Help
Replies
1
Views
2K
  • Introductory Physics Homework Help
Replies
1
Views
3K
Replies
1
Views
2K
Back
Top